Inscription / Connexion Nouveau Sujet
Niveau Maths sup
Partager :

question de topologie

Posté par san (invité) 04-01-07 à 21:50

Salut tout le monde
ma question est la suivante si A et B sont des fermées est ce que A+B est un fermé sinon j'aimerais qu'on m'aide à trouver un contre exemple et merci

Posté par
mouss33
re : question de topologie 04-01-07 à 21:54

si A et B sont fermé ET A est borné alors A+B est fermé

si tu n'as pas l'hypothese que A est borné, tu peux te servir de {k+1/k, k supérieur ou égale a 2} pour un contre exmple

Posté par
Cauchy
re : question de topologie 04-01-07 à 22:00

Salut,

on est dans R?

Posté par
mouss33
re : question de topologie 04-01-07 à 22:03

oui effectivment, ce que j'ai dit marche dans R!

Posté par san (invité)re : question de topologie 04-01-07 à 22:03

on est dans le cas géneral

Posté par
Cauchy
re : question de topologie 04-01-07 à 22:04

Dimension finie?

Posté par san (invité)re : question de topologie 04-01-07 à 22:06

A et B sont des parties quelconques d'un evn  

Posté par
mouss33
re : question de topologie 04-01-07 à 22:07

donc en dimension fini!

Posté par san (invité)re : question de topologie 04-01-07 à 22:08

il n'y a pas de dimension finie

Posté par
Cauchy
re : question de topologie 04-01-07 à 22:09

Bon de toute facon l'exo ici c'est juste de trouver un contre-exemple.

Mais je crois me souvenir dans le cas general que A+B ferme si l'un des deux est compact donc en dimension finie ca revient à borné.

Posté par
mouss33
re : question de topologie 04-01-07 à 22:12

ouais c'est ce que j'ai dit audébut!

Posté par
Cauchy
re : question de topologie 04-01-07 à 22:14

Oui

Posté par san (invité)re : question de topologie 04-01-07 à 22:21

oui il faut un contre exemple

Posté par san (invité)re : question de topologie 04-01-07 à 22:24

je n'ai pas saisie votre contre exemple Monsieur mouss33
si vous pouvez m'expliquer davantage ça serai mieux et désolé pour le dérangement

Posté par san (invité)re : question de topologie 05-01-07 à 13:10

quelqu'un peut il m'aider ?

Posté par san (invité)re : question de topologie 05-01-07 à 16:22

?????????????

Posté par
Camélia Correcteur
re : question de topologie 05-01-07 à 16:35

Bonjour
L'exemple consiste à prendre dans A tous les k+1/k avec k entier supérieur à 2 et B tous les -n pour n entier supérieur à 2. A et B sont fermés, mais comme 1/k=(k+1/k)+(-k) est dans A+B pour tout k supérieur à 2 et comme 0 n'y est pas, A+B n'est pas fermé.

Posté par san (invité)re : question de topologie 06-01-07 à 12:31

oui merci beaucoup camelia maintenant c'est clair



Vous devez être membre accéder à ce service...

Pas encore inscrit ?

1 compte par personne, multi-compte interdit !

Ou identifiez-vous :


Rester sur la page

Inscription gratuite

Fiches en rapport

parmi 1675 fiches de maths

Désolé, votre version d'Internet Explorer est plus que périmée ! Merci de le mettre à jour ou de télécharger Firefox ou Google Chrome pour utiliser le site. Votre ordinateur vous remerciera !